Figure It Out.

Geometry Level 4

Find the sum of all k k such that such a circle C 3 C_3 could exist in the following arrangement:

Two circles C 1 C_1 , C 2 C_2 of radii 1 3 \large\ \frac {1}{3} and 2 7 \large\ \frac {2}{7} respectively touch each other externally and they both touch a unit circle C C internally. A circle C 3 C_3 of radius R = 2 k R = \frac{2}{k} is inscribed to touch the circles C 1 , C 2 C_1, C_2 externally and the circle C C internally.


The answer is 22.

This section requires Javascript.
You are seeing this because something didn't load right. We suggest you, (a) try refreshing the page, (b) enabling javascript if it is disabled on your browser and, finally, (c) loading the non-javascript version of this page . We're sorry about the hassle.

1 solution

Aaghaz Mahajan
Nov 19, 2018

A very simple application of Kissing Circle's theorem........!!!

@Aaghaz Mahajan , whats that?

Priyanshu Mishra - 2 years, 6 months ago

Log in to reply

Here..........see this link...........
https://brilliant.org/wiki/descartes-theorem/

Aaghaz Mahajan - 2 years, 6 months ago

Log in to reply

Yes, that is the theorem to use, but it yields a value of R = 2 3 R = \dfrac{2}{3} and not 2 19 \dfrac{2}{19} .

Brian Charlesworth - 2 years, 6 months ago

Log in to reply

@Brian Charlesworth Umm, no sir........you might have taken the other value...........2/3 is the other value we get after solving the quadratic equation.........and to see which one is the correct value, we need to take into account the sign of the respective curvatures of the circles..........

Aaghaz Mahajan - 2 years, 6 months ago

Log in to reply

@Aaghaz Mahajan Hmmm... Yes, the sign is critical, and with the exterior unit circle ascribed a curvature of k 1 = 1 k_{1} = -1 then the Descartes' formula is satisfied using values k 2 = 3 , k 3 = 3.5 k_{2} = 3, k_{3} = 3.5 and k 4 = 1.5 k_{4} = 1.5 .

Brian Charlesworth - 2 years, 6 months ago

Log in to reply

@Brian Charlesworth Yes sir, exactly!!! I think that was the mistake you might have committed earlier......

Aaghaz Mahajan - 2 years, 6 months ago

Log in to reply

@Aaghaz Mahajan O.k., but the three circles C 1 , C 2 , C 3 C_{1}, C_{2}, C_{3} inside C C will then have respective radii of 1 3 , 2 7 \dfrac{1}{3}, \dfrac{2}{7} and 2 3 \dfrac{2}{3} . I don't know where the value of 2 19 \dfrac{2}{19} comes from; with the radii I have mentioned the other circle touching each of C 1 , C 2 , C 3 C_{1}, C_{2}, C_{3} has a radius of 1 17 \dfrac{1}{17} , (the second solution to the Descartes' formula).

Brian Charlesworth - 2 years, 6 months ago

Log in to reply

@Brian Charlesworth Ummm...........Sir, I don't se where you are going wrong.........this is a simple quadratic equation........Here.......The values of the respective curvatures are 3 , 3.5 and -1........the curvature which we need to find can be taken as x........Now, after applying Descarte's formula, we solve the equation to get the value of x as 9.5 and 1.5.........Now, we see that the circle with curvature 9.5 satisfies the constraint........and hence the radius is 1/(9.5) = 2/19............

Aaghaz Mahajan - 2 years, 6 months ago

@Brian Charlesworth Well in response to my previous comment, I notice the flaw in the question.............there are actually TWO values of the radii, and there exist TWO unique circles as a solution the problem (drawing a diagram helps in visualising it)........ I am sorry for any trouble caused.......

Aaghaz Mahajan - 2 years, 6 months ago

Log in to reply

@Aaghaz Mahajan Oh right, I see that now! So Priyanshu will have to specify which circle he means by C 3 C_{3} , since at the moment both 2 3 \dfrac{2}{3} and 2 19 \dfrac{2}{19} are possible radii.

Brian Charlesworth - 2 years, 6 months ago

Log in to reply

@Brian Charlesworth Yeah........I have filed a report........

Aaghaz Mahajan - 2 years, 6 months ago

0 pending reports

×

Problem Loading...

Note Loading...

Set Loading...